Difference between revisions of "2006 AMC 10B Problems/Problem 18"

m (added link to previous and next problem)
m (added category)
Line 25: Line 25:
  
 
*[[2006 AMC 10B Problems/Problem 19|Next Problem]]
 
*[[2006 AMC 10B Problems/Problem 19|Next Problem]]
 +
 +
[[Category:Introductory Algebra Problems]]

Revision as of 21:32, 3 August 2006

Problem

Let $a_1 , a_2 , ...$ be a sequence for which

$a_1=2$ , $a_2=3$, and $a_n=\frac{a_{n-1}}{a_{n-2}}$ for each positive integer $n \ge 3$.

What is $a_{2006}$?

$\mathrm{(A) \ } \frac{1}{2}\qquad \mathrm{(B) \ } \frac{2}{3}\qquad \mathrm{(C) \ } \frac{3}{2}\qquad \mathrm{(D) \ } 2\qquad \mathrm{(E) \ } 3$

Solution

Looking at the first few terms of the sequence:

$a_1=2 , a_2=3 , a_3=\frac{3}{2}, a_4=\frac{1}{2} , a_5=\frac{1}{3} , a_6=\frac{2}{3} , a_7=2 , a_8=3 , ....$

Clearly, the sequence repeats every 6 terms.

Since $2006 \equiv 2\bmod{6}$,

$a_{2006} = a_2 = 3 \Rightarrow E$

See Also